4
$\begingroup$

Consider the space of $n \times n$ matrices over the finite field $GF(2)$. Is it possible to choose $k$ matrices $A_i, i = 1 ,\cdots k$, such that for every non-zero vector $b \in \{0,1\}^k$ the matrix $\sum_{i=1}^k (b)_i A_i$ (with $(b)_i$ being the $i$-th entry of $b$) is full-rank? What is the largest $k$ for which this is possible? Is this possible for $k=n$?

I found that for $k=n=2$ we can simply take $A_1 = \left[\begin{matrix} 1&0\\ 0&1\end{matrix}\right]$ and $A_2 = \left[\begin{matrix} 1&1\\ 1&0\end{matrix}\right]$. Moreover, it is not hard to show that $k=2$ is the largest $k$ for which this is possible in $\{0,1\}^{2\times 2}$ (there are only 6 full-rank matrices in $\{0,1\}^{2\times 2}$ with the field $GF(2)$, after all). Similar matrices can be used for $k=2$ when $n > 2$, but I haven't found anything else. I feel like like the case $k=n$ should be possible in general, or at the very least $k=\lfloor n/2\rfloor$. but I haven't found valid matrices for $k=3$ and $n > 2$.

Thanks in advance.

$\endgroup$
1
  • 3
    $\begingroup$ It is not possible to have $k > n$, since in this case the set of first rows is linearly dependent, some linear combination will have the zero vector in the first row. $\endgroup$ Commented Jun 5 at 17:58

1 Answer 1

6
$\begingroup$

For $k\leqslant n$ such $k$ matrices exist. Take an irreducible polynomial $p$ of degree $n$, its companion matrix $A$ (a matrix with characterstic polynomial $p$) and $k$ matrices $A^i$ for $i=0,\dots,k-1$. Put $g(t)=\sum b_i t^i$. Recall that $p(A)=0$ by Hamilton -- Cayley. Since $0<\deg g\leqslant k-1<n$, the polynomials $p, g$ are coprime, thus, there exist the polynomials $f_1,f_2$ for which $f_1(t)p(t)+f_2(t)g(t)\equiv 1$. Then, $f_2(A)g(A)=f_1(A)p(A)+f_2(A)g(A)=I$, so, $g(A)$ is invertible.

But $n+1$ or more matrices do not exist: for every $n+1$ matrices there exist their non-trivial linear combination for which the first column is zero.

$\endgroup$
4
  • $\begingroup$ Would mind explaining a bit how this construction works? Also, would the same construction work for k < n? Thanks! $\endgroup$ Commented Jun 5 at 20:03
  • $\begingroup$ Of course every subset of a working set of matrices also works itself $\endgroup$ Commented Jun 5 at 20:22
  • $\begingroup$ Sorry, I meant if you used an irreducible polynomial of degree k < n. I think p and g would still be coprime in that case. $\endgroup$ Commented Jun 5 at 20:26
  • 3
    $\begingroup$ Another high-level explanation (which boils down to the same thing, but was helpful for me): this is precisely the left regular representation of the field extension GF(2^n) acting on itself, viewed as an n-dimensional vector space (hence, algebra) over GF(2). The nonzero matrices in the space being invertible corresponds to the fact that this is a field. $\endgroup$ Commented Jun 6 at 2:45

You must log in to answer this question.

Start asking to get answers

Find the answer to your question by asking.

Ask question

Explore related questions

See similar questions with these tags.